LSAT and Law School Admissions Forum

Get expert LSAT preparation and law school admissions advice from PowerScore Test Preparation.

 Administrator
PowerScore Staff
  • PowerScore Staff
  • Posts: 8917
  • Joined: Feb 02, 2011
|
#40045
Complete Question Explanation
(The complete setup for this game can be found here: lsat/viewtopic.php?t=6800)

The correct answer choice is (D)

If V is second, it follows that S must be first (from the first rule):
PT73_Game_#1_#6_diagram 1.png
Since T cannot be earlier than S and R, it must be later than both of them, in compliance with the second rule:
PT73_Game_#1_#6_diagram 2.png
Now, let’s examine the available options for W. According to the last rule, W must be earlier than both R and T, or else it must be later than both R and T. Either option can be satisfied in the setup above, which is why there are two possible solutions to this question:
PT73_Game_#1_#6_diagram 3.png
These two diagrams can be created even more quickly if you had access to the four templates discussed in the beginning, as only Templates 2A and 2B allow V to be the second piece on the CD:
PT73_Game_#1_#6_diagram 4.png
Answer choice (A): This answer choice cannot be true, because S must be first.

Answer choice (B): This answer choice cannot be true, because S must be first.

Answer choice (C): This answer choice cannot be true, because T must be either fourth or fifth.

Answer choice (D): This is the correct answer choice, as shown in the first local diagram above.

Answer choice (E): This answer choice cannot be true, because R must be either third or fourth.
You do not have the required permissions to view the files attached to this post.

Get the most out of your LSAT Prep Plus subscription.

Analyze and track your performance with our Testing and Analytics Package.